New Home Forums AMC 10 AMC 10A, 2019, Problem 24, Question and solution

Viewing 2 posts - 1 through 2 (of 2 total)
  • Author
    Posts
  • #99855
    Deepan Dutta
    Participant

    Let \(p\), \(q\), and \(r\) be the distinct roots of the polynomial \(x^3 - 22x^2 + 80x - 67\). It is given that there exist real numbers \(A\), \(B\), and \(C\) such that\[\dfrac{1}{s^3 - 22s^2 + 80s - 67} = \dfrac{A}{s-p} + \dfrac{B}{s-q} + \frac{C}{s-r}\]for all \(s\not\in\{p,q,r\}\). What is \(\tfrac1A+\tfrac1B+\tfrac1C\)?

    #99856
    Deepan Dutta
    Participant

    \(s^3 - 22s^2 + 80s - 67\) should be factorised as \((s-p)(s-q)(s-r)\), as \(p,q\) and \(r\) are the roots of the polynomial \(x^3 - 22x^2 + 80x - 67\).
    So, \(1=A(s-q)(s-r)+B(s-r)(s-p)+C(s-p)(s-q)\)

    Now for \(s=p\), we would get, \(\frac{1}{A}=(p-q)(p-r)\)
    Similarly, for \(s=q\), we get, \(\frac{1}{B}=(q-p)(q-r)\)
    And, for \(s=r\), we get, \(\frac{1}{C}=(r-p)(r-q)\)
    \(\therefore\) Adding them we can get, \(\frac{1}{A}+\frac{1}{B}+\frac{1}{C}=p^2+q^2+r^2-pq-qr-rp\)
    Now form the given equation we can obtain the sum of the roots,
    \(p+q+r=22\)
    Also the sum of the roots taken two at a time \(pq+pr+rq=80\)
    So, \(p^2+q^2+r^2=(p+q+r)^2-2(pq+pr+rq)\)
    \(\Rightarrow p^2+q^2+r^2=22^2-160\)
    \(\Rightarrow p^2+q^2+r^2=484-160=324\)
    So, \(\frac{1}{A}+\frac{1}{B}+\frac{1}{C}=324-80=\boxed{244}\)

    • This reply was modified 1 year, 3 months ago by Deepan Dutta.
Viewing 2 posts - 1 through 2 (of 2 total)
  • You must be logged in to reply to this topic.
linkedin facebook pinterest youtube rss twitter instagram facebook-blank rss-blank linkedin-blank pinterest youtube twitter instagram